- PowerScore Staff
- Posts: 5972
- Joined: Mar 25, 2011
- Mon Jun 25, 2018 9:04 am
#88285
Complete Question Explanation
(The complete setup for this game can be found here: lsat/viewtopic.php?f=176&p=88283#p88283)
The correct answer choice is (A).
The question stem specifies that both X and Y confess. Thus, neither X nor Y can be questioned on day four because the suspect questioned on day four does not confess. Of course, during the setup discussion we determined that Z cannot be questioned on day four, so the only suspect that can be questioned on day four is V, and accordingly V does not confess. Thus, answer choice (A) cannot occur and is correct.
With the other answer choices, consider that from the fourth rule X cannot be questioned first, and the suspect questioned on day four does not confess, leaving only days two and five as available days where X could be questioned and confess. Thus, answer choice (B) could be true and is incorrect.
Y could be questioned on either day one, day two, or day five, and so answer choice (C) could be true and is incorrect.
Z could be questioned on day one or two, and the suspect questioned on each of those days can either confess or not confess, and so answer choices (D) and (E) are both possible, and thus incorrect.
(The complete setup for this game can be found here: lsat/viewtopic.php?f=176&p=88283#p88283)
The correct answer choice is (A).
The question stem specifies that both X and Y confess. Thus, neither X nor Y can be questioned on day four because the suspect questioned on day four does not confess. Of course, during the setup discussion we determined that Z cannot be questioned on day four, so the only suspect that can be questioned on day four is V, and accordingly V does not confess. Thus, answer choice (A) cannot occur and is correct.
With the other answer choices, consider that from the fourth rule X cannot be questioned first, and the suspect questioned on day four does not confess, leaving only days two and five as available days where X could be questioned and confess. Thus, answer choice (B) could be true and is incorrect.
Y could be questioned on either day one, day two, or day five, and so answer choice (C) could be true and is incorrect.
Z could be questioned on day one or two, and the suspect questioned on each of those days can either confess or not confess, and so answer choices (D) and (E) are both possible, and thus incorrect.
Dave Killoran
PowerScore Test Preparation
Follow me on X/Twitter at http://twitter.com/DaveKilloran
My LSAT Articles: http://blog.powerscore.com/lsat/author/dave-killoran
PowerScore Podcast: http://www.powerscore.com/lsat/podcast/
PowerScore Test Preparation
Follow me on X/Twitter at http://twitter.com/DaveKilloran
My LSAT Articles: http://blog.powerscore.com/lsat/author/dave-killoran
PowerScore Podcast: http://www.powerscore.com/lsat/podcast/